Difference between revisions of "2024 AMC 10B Problems/Problem 24"

(Solution)
Line 3: Line 3:
 
==Problem==
 
==Problem==
  
==Solution==
+
==Solution #1==
 +
Let x,y,z be 3 sides of triangle, r = <math>\frac{s}{p} </math>, 2p = x+y+z , 2s = ax=by=cz =2pr
 +
<cmath>s= rs( \frac{1}{a} + \frac{1}{b} +\frac{1}{c} ) </cmath>
 +
<cmath> r( \frac{1}{a} + \frac{1}{b} +\frac{1}{c} ) = 1 </cmath>
 +
<cmath>r =1,2,3 </cmath>
 +
case r=1:
 +
<cmath>( \frac{1}{a} + \frac{1}{b} +\frac{1}{c} ) = 1 </cmath>
 +
given that 1<=a<=b<=c<=9
 +
<cmath> 1 = ( \frac{1}{a} + \frac{1}{b} +\frac{1}{c} )  <= \frac{3}{a} </cmath>
 +
<cmath> a <= 3 </cmath>
 +
case 1.1 no solution for b,c <cmath>  a= 2 ,  ( \frac{1}{2} + \frac{1}{b} +\frac{1}{c} ) = 1  </cmath> 
 +
case 1.2 <cmath>  a= 3 ,  ( \frac{1}{3} + \frac{1}{b} +\frac{1}{c} ) = 1 , (b,c) =(3,3) </cmath>
 +
case r=2:
 +
<cmath>( \frac{1}{a} + \frac{1}{b} +\frac{1}{c} ) = \frac{1}{2} </cmath>
 +
<cmath> (a,b,c) =(6,6,6) </cmath>
 +
case r=3:
 +
<cmath>( \frac{1}{a} + \frac{1}{b} +\frac{1}{c} ) = \frac{1}{3} </cmath>
 +
<cmath> (a,b,c) =(9,9,9) </cmath>
 +
answer  <math>\boxed{\textbf{(B) } 3}</math>
 +
 
 +
~[https://artofproblemsolving.com/wiki/index.php/User:Cyantist luckuso]
  
 
==See also==
 
==See also==

Revision as of 01:41, 14 November 2024

The following problem is from both the 2024 AMC 10B #24 and 2024 AMC 12B #18, so both problems redirect to this page.

Problem

Solution #1

Let x,y,z be 3 sides of triangle, r = $\frac{s}{p}$, 2p = x+y+z , 2s = ax=by=cz =2pr \[s= rs( \frac{1}{a} + \frac{1}{b} +\frac{1}{c} )\] \[r( \frac{1}{a} + \frac{1}{b} +\frac{1}{c} ) = 1\] \[r =1,2,3\] case r=1: \[( \frac{1}{a} + \frac{1}{b} +\frac{1}{c} ) = 1\] given that 1<=a<=b<=c<=9 \[1 = ( \frac{1}{a} + \frac{1}{b} +\frac{1}{c} )  <= \frac{3}{a}\] \[a <= 3\] case 1.1 no solution for b,c \[a= 2 ,  ( \frac{1}{2} + \frac{1}{b} +\frac{1}{c} ) = 1\] case 1.2 \[a= 3 ,  ( \frac{1}{3} + \frac{1}{b} +\frac{1}{c} ) = 1 , (b,c) =(3,3)\] case r=2: \[( \frac{1}{a} + \frac{1}{b} +\frac{1}{c} ) = \frac{1}{2}\] \[(a,b,c) =(6,6,6)\] case r=3: \[( \frac{1}{a} + \frac{1}{b} +\frac{1}{c} ) = \frac{1}{3}\] \[(a,b,c) =(9,9,9)\] answer $\boxed{\textbf{(B) } 3}$

~luckuso

See also

2024 AMC 10B (ProblemsAnswer KeyResources)
Preceded by
Problem 23
Followed by
Problem 25
1 2 3 4 5 6 7 8 9 10 11 12 13 14 15 16 17 18 19 20 21 22 23 24 25
All AMC 10 Problems and Solutions
2024 AMC 12B (ProblemsAnswer KeyResources)
Preceded by
Problem 17
Followed by
Problem 19
1 2 3 4 5 6 7 8 9 10 11 12 13 14 15 16 17 18 19 20 21 22 23 24 25
All AMC 12 Problems and Solutions

The problems on this page are copyrighted by the Mathematical Association of America's American Mathematics Competitions. AMC logo.png